Essayist: If Earth's population continues to grow geometrically, then in a few centuries there will be ten people for...

on January 7, 2020

Narrowing down conclusion

I actually had a bit of trouble narrowing this down, but ultimately looked after the however; conclusion indicator. Could you please explain why each of them are incorrect/correct? Thank you!

Reply
Create a free account to read and take part in forum discussions.

Already have an account? log in

Annie on January 13, 2020

Hi @mamie,

This question asks you to pick the answer choice which "most accurately expresses" the conclusion in the argument. When you get a question like this, you should note the wording of the question stem, as this means that there may be multiple answer choices that "express" some version of the conclusion, but you're just looking for the best one.

Before turning to the answer choices, try to determine what the conclusion of the argument is for yourself. As you picked out, the conclusion is that planetary travel would be a "temporary solution."

Answer Choices:
Answer (A) is incorrect because this is just one of the facts provided. You're looking for the conclusion of the argument, which happens later. This just is laying the background for the discussion.

Answer (B) is incorrect but a close one. It is incorrect because the essayist never states that the geometric growth of earth's population will continue, rather he says it may continue and that, if it does, planetary travel will not be a solution. Additionally, the argument is about how to solve the problem of overcrowding if it exists, not whether overcrowding will exist. So, this answer choice is off topic.

Answer (C) is incorrect because it simply restates a premise (the last sentence) offered in support of the conclusion. This is not the conclusion of the argument, but rather an example used to support the conclusion.

Answer (D) is incorrect because, like described above, the essayist never states that the earth's population will continue to grow, but rather treats that as a hypothetical.

Answer (E) is correct because it restates the conclusion you identified. The conclusion is that "this would be, however, a temporary solution" and this answer choice simply takes it a step further by explaining what "this" refers to.